Q21

 
topcow500
Thanks Received: 0
Forum Guests
 
Posts: 6
Joined: August 23rd, 2013
 
 
 

Q21

by topcow500 Fri Aug 23, 2013 2:12 pm

Question 21:

I don't understand how the answer to this question is (C), or "four." I've gone over it multiple times. It doesn't make sense according to the wording of the question. It reads:

"If Pavlovich and Tsudama are teammates, then for how many of the individuals can it be exactly determined where his or her team places?"

The key word here is "exactly."

Tsudama is on the second place team, so Pavlovich is also on the second place team.

Sethna is on the team from Gillom High, which places higher than the team from Hilltop High, so we can infer that Sethna is on the first place team (since S does not fit in the second place team, and cannot be last place). Easy enough. Also Gillom is first place, but that doesn't really matter.

Since Pavlovich places higher than Navarro, then Navarro must be on the third place team.

Now we know that S is on the first place team, and N is on the third place team. Thus, I answered (A) to this question, which is "two." The "correct" answer is "four."

Problem:
How can it be EXACTLY determined where Mei and O'Rourke place? In this case it seems that they are interchangeable between the first- and third-place teams. Mei is not on the same team as Pavlovich; this does not place Mei first or third. O'Rourke is a random; same deal. Therefore, in this situation their placement doesn't matter, and as such cannot be EXACTLY determined.

I truly believe this is a mistake by the authors. I don't understand why nobody else on here has had a problem with this. It's the only one I got wrong!
 
fmuirhea
Thanks Received: 64
Forum Guests
 
Posts: 46
Joined: November 29th, 2012
 
This post thanked 1 time.
 
 

Re: Problem with the "correct" answer choice.

by fmuirhea Fri Aug 23, 2013 8:05 pm

You're going to smack your head when I reveal this: you forgot to count T and P as being exactly determined. Your analysis is spot on; O/M are indeed able to switch off between teams 1 and 3.

1: G with S & O/M
2: F/H with T & P
3: F/H with N & O/M

The wording of the question does not imply "in addition to T and P," or "besides T and P," so you need to include them in the count.
 
topcow500
Thanks Received: 0
Forum Guests
 
Posts: 6
Joined: August 23rd, 2013
 
 
 

Re: Problem with the "correct" answer choice.

by topcow500 Sat Aug 24, 2013 3:42 pm

fmuirhea Wrote:You're going to smack your head when I reveal this: you forgot to count T and P as being exactly determined. Your analysis is spot on; O/M are indeed able to switch off between teams 1 and 3.

1: G with S & O/M
2: F/H with T & P
3: F/H with N & O/M

The wording of the question does not imply "in addition to T and P," or "besides T and P," so you need to include them in the count.


Ha! Wow thank you SO much! Yes, there is a palm-sized red mark on my forehead now. What I call "over assuming" has been problematic for me on the logic games. I took for granted that T comes in second place as though nobody would forget that rule. My apologies for my heretic remarks. The LSAT writers remain infallible...
 
JenniferK632
Thanks Received: 0
Jackie Chiles
Jackie Chiles
 
Posts: 43
Joined: January 18th, 2020
 
 
 

Re: Q21

by JenniferK632 Thu Sep 10, 2020 9:58 am

Whenever games have the "how many options will be decided" question, I find it really hard to check my work. I just end up filling in the conditional, or going through the rules and looking for limitations, then praying for the best.

Any foolproof way to check your work here?
 
Laura Damone
Thanks Received: 94
Atticus Finch
Atticus Finch
 
Posts: 468
Joined: February 17th, 2011
 
 
 

Re: Q21

by Laura Damone Wed Oct 28, 2020 1:23 pm

Please forgive the delayed response here. We had a technical issue that caused this post to get buried!

These questions are all about following the Inference chain! If it's a conditional question, plug in the new rule and figure out what must be true based on that rule. When you reach what you think is the end of the chain, ask yourself "who's left that could fill the remaining slots" and jot them down. Often in doing this, you'll see another inference!

If the question isn't conditional, it's just asking about who is definitively placed in your master diagram. If you've placed some elements, go back and double check that there are no more definite placements by taking a second pass through the rules and asking "who's left" as advised above. If you haven't placed any elements in your master yet, start at square one and work your way through each rule and its inferences again until you figure out some definites. Unless, of course, Zero is an answer :)

Hope this helps!
Laura Damone
LSAT Content & Curriculum Lead | Manhattan Prep